LSAT and Law School Admissions Forum

Get expert LSAT preparation and law school admissions advice from PowerScore Test Preparation.

 Administrator
PowerScore Staff
  • PowerScore Staff
  • Posts: 8917
  • Joined: Feb 02, 2011
|
#72958
Complete Question Explanation

Justify the Conclusion. The correct answer choice is (E).

The argument presents several conditional claims with no qualifiers - no element of probability, just absolute certainty. The author is completely certain, based on these conditional claims, that " building the convention center will increase the city’s tax revenues" (the last sentence). The stem asks us to justify (prove) this conclusion, which means despite how certain the author sounded, there must be something missing from his argument, something which when we supply it will complete the conditional chain and guarantee the truth of the final sentence.

The conditional premises are as follows:

BCC (Build Convention Center) :arrow: SNOC (Several National Organization Conventions)
SLC (Several Large Conventions) :arrow: VI (Visitors Increase)
VI :arrow: TRI (Tax Revenues Increase)

The conclusion is:

BCC :arrow: TRI

At this point we should notice that there is a slight shift between the first and second premises, from SNOC to SLC, and that is the gap which, once we close it, will get us all the way from the first sufficient condition to the final necessary condition. Our prephrase must be along the lines of "if there are national organization conventions, they will be large."

Answer choice (A): VI :arrow: TRI, the negation of the final premise, does not connect the conditions in our chain and does not match our prephrase. Loser.

Answer choice (B): VI :arrow: MSVI (money spent by visitors increases) brings up a new condition and does not connect the building of the convention center to increased tax revenues, the conclusion we are seeking to justify.

Answer choice (C): TRI :arrow: BCC is a reversal of the conclusion, which does nothing. A contrapositive would have done the job, but this is not that.

Answer choice (D): BCC :arrow: RVC (regular visitors continue) brings up a new condition and does not connect the building of the convention center to increased tax revenues.

Answer choice (E): This is the correct answer choice. SNOC :arrow: SLC closes the gap and matches our prephrase perfectly. We now have this complete conditional chain:

BCC :arrow: SNOC :arrow: SLC :arrow: VI :arrow: TRI, and it must be true that BCC :arrow: TRI. Winner!
 mpoulson
  • Posts: 148
  • Joined: Mar 25, 2016
|
#25813
Hello,

Can you explain why the answer is E? I originally chose B because if it wasn't true then it seems to undermine the argument completely. I don't clearly see why E is more effective. Thank you.

- Micah
 Nikki Siclunov
PowerScore Staff
  • PowerScore Staff
  • Posts: 1362
  • Joined: Aug 02, 2011
|
#25890
Hi Micah,

This is not an Assumption but a Justify question, so your strategy for validating the correct answer choice is incorrect. In Justify stimuli, the correct answer choice - when added to the premises - must prove the conclusion.

With that in mind, please provide a detailed breakdown of how you understood the argument. Deconstruct the stimulus into premises/conclusion. Then explain to us whether the conclusion logically follows from the premises, and if not - why not? Make sure to tell us how you prephrased the answer you were looking for. Finally, please explain what specifically made the (incorrect) answer choice you chose particularly attractive.

The more you tell us about your method of approach (which was clearly incorrect in this case, as you conflated an Assumption with a Justify question), the better we can help you figure it out.

Thanks!
 cpando1995@gmail.com
  • Posts: 18
  • Joined: Jul 07, 2016
|
#30487
Hi Powerscore Staff,

I had much difficulty with this question and, when I timed myself, I originally chose C. I then went back and tried breaking the problem down and chose D. During my second try at it, I focused on how one premise said "total number of visitors" but another premise said "number of visitors. I can see now how the part that was missing was the connection between "conventions" and "large conventions", but that only became apparent once I saw the right answer. Can anyone explain how I should have approached this question and why my previous focus on visitors was misguided?



Thank you
User avatar
 Jonathan Evans
PowerScore Staff
  • PowerScore Staff
  • Posts: 726
  • Joined: Jun 09, 2016
|
#30542
CPando,

Here's how to attack this question:
  1. Notice the kind of language involved in the stimulus. You should remark immediately that every statement is a strong conditional.
  2. Next, remark the clarity of the logical structure. There is little to no ambiguity involved. You should note that the use of stimuli such as these is strongly correlated with formal logic justify the conclusion or must be true scenarios.
  3. ID the conclusion or claim.
  4. Note the evidence that backs it up.
  5. Describe the train of thought that leads from the premises to the conclusion.
  6. Note any obvious logical flaws. If you don't notice any, don't worry.
  7. Read and understand the question: Justify the Conclusion. You are looking for information sufficient to guarantee the validity of the conclusion. In other words, given the information already provided, you must add one more statement that will enable the given information to be sufficient to imply the conclusion. You must connect the left hand side of the arrow to the right hand side
  8. Diagram if it is helpful to you. Keep your symbolization simple and consistent. Now note any gaps you may have missed that occur while you are diagramming
Here's the diagram:
  • [P]BCC :arrow: Conv
    [P]LConv :arrow: V
    [P]V :arrow: TR
    [C]BCC :arrow: TR
In a Justify question involving conditionals, you must connect the sufficient side of the conclusion to the necessary side of the conclusion. You do this with the information contained in the premises along with the information contained in the credited response. Run through the sequence to see where it breaks down. The place where it breaks down is what you use to come up with a prephrase.

Let's start with the sufficient term of the conclusion, "BCC":
  • [P]BCC :arrow: Conv
    [P]LConv :arrow: V
Stop. Here's where things broke down. You can't get from Conv to LConv. This is a Shell Game language shift. You can follow the premises the rest of the way all the way to TR, but you must connect Conv to LConv. Further you know that you must start from the left and arrive at the right (Conv :arrow: LConv). Otherwise you will have a Mistaken Reversal.

The only answer choice that even has these terms is (E), so you're done. Double check and indeed (E) translates to Conv :arrow: LConv.

Notice the importance of keeping precise, succinct, consistent terms in any symbolizations. Notice also the importance of running through the reasoning on your own before diving into symbols. Without understanding the structure yourself, symbols are much less useful, and you're prone to making mistakes.
 cpando1995@gmail.com
  • Posts: 18
  • Joined: Jul 07, 2016
|
#30555
Thank you so much for the explanation; it definitely makes questions like this one less intimidating.
 medialaw111516
  • Posts: 80
  • Joined: Dec 11, 2018
|
#72052
Hi, is B wrong because of "increase?" I initially understood it as, they have to spend money in order for the tax revenue to increase, but upon reading it for review, I see that B is actually saying visitors will spend more money that they did in the past, which isn't necessary true.
 Paul Marsh
PowerScore Staff
  • PowerScore Staff
  • Posts: 290
  • Joined: Oct 15, 2019
|
#72292
Hi Medialaw! For Justify questions, we are looking for an answer choice that will cover the gap between the conclusion and premises in order to make the conclusion totally, 100% follow from the premises.

In this stimulus, a series of conditional statements is used to make an argument. The conditionals link up nicely to form one long conditional chain of an argument - with one exception: just because several national professional organizations will hold conventions there doesn't mean that those conventions will be large! Jonathan outlined this beautifully up above with a great diagram of the conditional statements. Once you are comfortable with conditional logic, that "gap" between conventions and large conventions will jump out at you. For Justify questions like this that use conditional logic, you want be able to Pre-Phrase the answer almost exactly. So by the time you're first looking at the answer choices, you want to be exclusively scanning for something about how the conventions will be large. If an answer choice doesn't address large conventions, it's not even worth considering here because it doesn't deal with the gap in the conditional logic.

Answer Choice (E) addresses that gap perfectly. Answer Choice (B), on the other hand, does not. (B) starts talking about "money spent by visitors", which does not play any part in the stimulus; (B) therefore does nothing to make the conclusion follow logically from the premises. This question is really about how comfortable you are with conditional reasoning. Conditionals are a tricky concept for many LSAT students, so don't feel discouraged! Review Lesson 2 (and homework) in the Course Books, and keep practicing conditional drills and problems. Hope that helps!
User avatar
 Albertlyu
  • Posts: 98
  • Joined: Jul 18, 2020
|
#79934
thanks, this one almost drove me crazy when doing the first time. it felt like an itch I just could not scratch.
User avatar
 silver2731
  • Posts: 11
  • Joined: Mar 01, 2022
|
#94054
My reasoning behind why the answer is E is that the stimulus conveniently make a logical leap from conventions will be held to large conventions are held in the city. Tax revenue increase is dependent on the large conventions to be held but the premise doesn't guarantee if large conventions will be held. only just conventions.

The answer E fills the gap by saying the conventions that are stated to be held are indeed large.

If something like "some of those conventions will be large" would have made the E more easily detectible..

Get the most out of your LSAT Prep Plus subscription.

Analyze and track your performance with our Testing and Analytics Package.